PT 51, S1, Q10 Question Forum

Prepare for the LSAT or discuss it with others in this forum.
Post Reply
User avatar
JWP1022

Bronze
Posts: 269
Joined: Mon Dec 03, 2012 10:15 pm

PT 51, S1, Q10 Question

Post by JWP1022 » Mon Sep 02, 2013 1:58 pm

Manhattan Thread: http://www.manhattanlsat.com/forums/q10 ... 65c64aca40

Got this question right on my PT, but given the debate in the linked thread about choice A, does the conditional language actually matter? From my perspective, the stimulus only talks about lowering one's chances of developing cardiorespiratory illnesses "significantly." Answer choice A only talks about the argument failing to consider any old exercise lowering the risk in general, not significantly, which means it's not a flaw because it's not really in the scope of the stimulus. Is this a safe reason to eliminate question choice A? Just because other kinds of exercise can lower risk of CR disease does not also imply that they can lower the risk significantly, right?

Thanks!

User avatar
SilvermanBarPrep

Bronze
Posts: 434
Joined: Thu Aug 22, 2013 9:19 pm

Re: PT 51, S1, Q10 Question

Post by SilvermanBarPrep » Mon Sep 02, 2013 8:59 pm

Exactly right. In addition, more specifically, it's not a flaw that the author failed to consider the possibility that the risk of developing certain cardio-respiratory illnesses can be reduced by means other than exercise because the author never stated that exercise was the only method of reducing such illnesses. In other words, the author left open the possibility that means other than exercise exist and merely made the distinction between strenuous and non-strenous exercise in regards to reducing such illnesses.

The problem here is simply that the author is conclusion that the new studies are more valid than previous studies without first proving that such a conclusion is valid.

Post Reply

Return to “LSAT Prep and Discussion Forum”